2
$\begingroup$

I want to solve approximately the following variational problem:

Given a function $c:[-1,1]^2\rightarrow [0,1]$, constants $p_1...p_n\in \mathbb{R}^+$, $\alpha_1...\alpha_n\in \mathbb{R}$, and $\beta_1...\beta_n\in \mathbb{R}$ let

$$V =\sup_{f_i,g_i}\left\{\sum_{i=1}^n \;\;p_i\cdot \int_0^1\int_0^1 f_i(x,y)\cdot g_i(x,y)\; dx\; dy \;\;: \sum_{i}\alpha_i\cdot f_i = c = \sum_i \beta_i\cdot g_i \right\}$$

where the supremum is taken over all square integrable functions $f_i,g_i:[-1,1]^2\rightarrow [0,1]$.

Observe that since all functions have domain $[-1,1]^2$ and range $[0,1]$, the value of each integral $\int_{0}^1\int_{0}^1 f(x,y)\cdot g(x,y) \;dx\; dy$ lies between $0$ and $1$. Therefore $V$ lies between $0$ and $\sum_{i} p_i$.

Assumptions:

All functions $f_i$ and $g_i$ and $c$ can be square integrable.

If it makes the problem simpler, $c:[-1,1]^2\rightarrow [0,1]$ can be assumed to be $c(x,y)=1$ if $x=y$ and $c(x,y)=0$ otherwise.

Questions:

  1. What numerical methods are available to approximate the value of $V$ up to an $\varepsilon$ additive factor? In other words, I want to find a $V'$ such that $|V-V'|\leq \varepsilon$, where $\varepsilon$ is a given precision parameter.

  2. What is the rate of convergence of such methods?

$\endgroup$
8
  • $\begingroup$ Obs: this question was asked more than one month ago in scicomp, but I've got no answer, so I decided to ask it here. $\endgroup$
    – verifying
    Jan 17, 2016 at 21:33
  • $\begingroup$ You have to be a bit more precise. Are the constants $p_i$ positive or negative? You seem to suggest that they are positive. This needs to be clarified. Also, be very precise about the functions $f_i,g_i$ are the constraints $0\leq f_i, g_i\leq 1$ part of your assumptions? If so state this explicitly. In this case the problem reduces to maximizing a convex function over a convex set. The maxima, if they exist are located at one of the extremal points of the (convex) constraint set. $\endgroup$ Jan 17, 2016 at 21:54
  • $\begingroup$ Liviu thanks for your comment. Indeed, all constants given in the problem may be assumed to be non-negative. The functions $f_i,g_i$ are all from $[0,1]^2\rightarrow [0,1]$ and can be assumed to be square integrable. What are the methods to solve the convex optimization problem arising from this problem? $\endgroup$
    – verifying
    Jan 17, 2016 at 22:35
  • $\begingroup$ In the special case $c(x,y)=1$ if $x=y$ and $0$ otherwise, the functions $f_i, g_i$ must be equal to $0$ a.e. since they are $\geq 0$ and the constants are $\geq 0$. The functional to maximize is then identically zero. $\endgroup$ Jan 17, 2016 at 23:42
  • $\begingroup$ My mistake. Only p_i's are required to be positive. Also all functions are from $[-1,1]^2\rightarrow [0,1]$. Thanks for pointing out. $\endgroup$
    – verifying
    Jan 18, 2016 at 6:53

1 Answer 1

1
$\begingroup$

A bit long for a comment.

Let's clean up the formulation a bit:

  • First, the domain $[-1,1]^2$ of definition does play any role, and hence, we assume that all respective quantities are functions on some $\Omega$ (and keep in mind that $\Omega=[-1,1]^2$ is the desired case).

  • Then we see that we are supremizing over $f = (f_1,\dots,f_n)\in L^2(\Omega)^n$ and $g=(g_1,\dots,g_n)\in L^2(\Omega)^n$, so in total over $(f,g)\in L^2(\Omega)^{2n}$ which is a nice Hilbert space.

  • Now we introduce new variables $\tilde f_i = p_i f_i$ and observe that the objective is now $$\sum_i p_i\langle f_i,g_i\rangle = \sum_i \langle p_if_i,g_i\rangle = \sum_i \langle \tilde f_i,g_i\rangle = \langle \tilde f,g\rangle_{L^2(\Omega)^n}.$$

  • Let's turn to the constraints: You have the bounds $0\leq f_i,g_i\leq 1$ which turn into $0\leq g_i\leq 1$ and $0\leq \tilde f_i\leq p_i$. The equality constraints are of the form $Af=c$ with $$ A:L^2(\Omega)^n\to L^2(\Omega),\quad Af = \sum_i \alpha_i f_i$$ $$ B:L^2(\Omega)^n\to L^2(\Omega),\quad Bg = \sum_i \beta_i g_i.$$ This turns into $$Af = \sum_i \alpha_if_i = \sum_i \tfrac{\alpha_i}{p_i}p_if_i = \sum_i \tfrac{\alpha_i}{p_i}\tilde f_i = \tilde A\tilde f.$$

With all this, the problem is $$ \begin{array}{lrl} \sup\limits_{(\tilde f,g)\in L^2(\Omega)^{2n}} \langle \tilde f,g\rangle_{L^2(\Omega)^n} & 0&\leq \tilde f_i\leq p_i,\\ & 0&\leq g_i\leq 1,\\ & \tilde A\tilde f &= c,\\ & Bg &= c. \end{array} $$

The objective is not a convex function in $\tilde f$ and $g$ jointly but it's smooth. Hence, your problem classifies as a non-linear optimization problem with linear constraints.

$\endgroup$
1
  • $\begingroup$ Thanks for the reformulation! It is much cleaner indeed. $\endgroup$
    – verifying
    Jan 18, 2016 at 9:08

Your Answer

By clicking “Post Your Answer”, you agree to our terms of service and acknowledge you have read our privacy policy.

Not the answer you're looking for? Browse other questions tagged or ask your own question.